Suite récursive

Bonjour, on pose $\quad \displaystyle u_n=\frac{1}{n}\sum_{k=0}^{n-1} \frac{u_k}{n-k},\ $ avec $u_0=1$.
Peut-on trouver un équivalent simple de $u_n$.
Merci.
«1

Réponses

  • Bonjour,
    Numériquement il semble qu'on a $u_n \sim \frac{C}{n^2}$ avec $C\approx 5{,}2$.
    Autre remarque : $\forall x\in{]{-}1,1[},\ \sum\limits_{n=0}^\infty u_n x^n = e^{{\rm Li}_2(x)}$ mais je ne sais absolument pas si ça peut être utile.

    PS: AD, tu as fait une erreur en modifiant. C'était $u_n=\frac{1}{n}\sum_{k=0}^{n-1} \frac{u_k}{n-k}$.
    [En effet, il y a un $t$ qui s'était introduit ! :-S J'ai corrigé. AD]
  • Je reprends. On a $$n^2u_n =\sum_{k=0}^{n-1} u_k \frac{n}{n-k} \longrightarrow \sum_{k=0}^\infty u_k $$ par convergence monotone (la suite $(u_n)$ est évidemment positive). Or $\sum\limits_{k=0}^\infty u_k = e^{{\rm Li}_2(1)}=e^{\zeta(2)} \approx 5{,}181$. Donc $$\fbox{$u_n\sim \dfrac{e^{\zeta(2)}}{n^2}$}$$ Joli exercice !!(:D
  • J'ai en partie transgressé le principe qui veut qu'on donne des indices et pas la solution. Mais j'ai eu un peu de mal à trouver (j'avais largement sous-estimé le début de la somme et j'étais parti sur une mauvaise piste) et je ne dois pas être le seul car personne d'autre n'a répondu en sept heures ! Donc j'ai répondu de la même manière qu'aux défis du camarade etanche. Néanmoins, il te reste des choses à justifier mathspe, par exemple le fait que $\forall x\in{]{-}1,1[},\ \sum\limits_{n=0}^\infty u_n x^n = e^{{\rm Li}_2(x)}$ et $\sum\limits_{k=0}^\infty u_k = e^{{\rm Li}_2(1)}$ :-).

    PS: Je ne l'ai pas précisé, mais la définition du dilogarithme ${\rm Li}_2$ est : $\forall x\in[-1,1], \;{\rm Li}_2(x) := \sum\limits_{n=0}^\infty \frac{x^n}{n^2}$.
  • Bonjour Calli,

    Comment fais-tu pour trouver numériquement ce genre d’équivalents?
    Teste-tu les produits $n^k \times u_n$ ou y a-t-il moins « naïf »?
  • sevaus, j'utilise mes cours de physique-chimie ! (:D Je m'explique. En TP de physique-chimie, quand on voulait déterminer une loi du type $Y =c X^\alpha$ à partir de valeurs expérimentales, on traçait nos mesures $(X_i,Y_i)$ sur un papier millimétré en échelle log-log et on regardait la pente et l'ordonnée à l'origine de la droite obtenue, qui valent respectivement $\alpha$ et $\ln(c)$. Ici, j'ai fait pareil : j'ai tracé $-\ln(u_n)$ en fonction de $\ln(n)$. À partir de $n\geqslant 10$, on obtient très clairement une droite et j'ai mesuré sa pente : $\frac{-\ln(u_{10\,000})+\ln(u_{1000})}{\ln(10\,000)-\ln(1000)} \approx 2{,}00$. Ensuite, le facteur multiplicatif est l'exponentielle de moins l'ordonnée à l'origine de la droite, ou tout simplement $u_{10\,000}\times 10\,000^2$.
  • @Calli : excuse-moi mais ça sort d'où ça : ??::o
    $$
    \forall x\in{]{-}1,1[},\quad \sum\limits_{n=0}^\infty u_n x^n = e^{{\rm Li}_2(x)}$$
  • Totem, ce n'est pas un exercice facile que nous avons là. J'attends d'ailleurs toujours tes réponses ici : http://www.les-mathematiques.net/phorum/read.php?4,2092668,2093330#msg-2093330 :-D
  • @skyffer: oui effectivement , j'ai trouvé "par hasard" la correction de Chaurien sur le forum... alors...:-)
  • totem : J'ai passé sous silence un calcul non trivial. Si tu veux essayer de le faire, tu auras besoin des outils suivants :
    1. la formule de récurrence que vérifie $(u_n)$,
    2. le théorème de Fubini (oh ! encore lui (:P), mais avec les séries cette fois), édit : ou bien un produit de Cauchy
    3. savoir dériver et primitiver une série entière,
    4. et savoir résoudre une équation différentielle linéaire du premier ordre.
  • @Calli : je connais ces outils et je les maîtrise moyen / assez bien . Si tu me guides , ça peut marcher :-)

    Par contre la formule de récurrence pour $u_n$...bon ok il faut exprimer $u_{n+1}$ en fonction de $u_n$ j'y pense en l'écrivant !!
  • totem : Pas besoin de changer la formule de récurrence ; on garde $u_n$ en fonction de $u_0,u_1,\dots,u_{n-1}$. Je donne le point de départ. Soit $f:x\in{]-1,1[}\mapsto\sum\limits_{n=0}^\infty u_n x^n$. On commence par écrire : $$\forall x\in{]-1,1[},\quad f(x) = \sum_{n=0}^\infty \frac1n \sum_{k=0}^{n-1} \frac{u_k}{n-k} x^n$$ et ensuite on manipule les signes $\sum$ et leurs indices.
  • @Calli impressionnant !

    En ce qui me concerne je ne vois pas comment appliquer la convergence monotone ici : $\sum_{k=0}^{n-1} u_k \frac{n}{n-k} \longrightarrow \sum_{k=0}^\infty u_k$, la suite $(u_k \frac{n}{n-k})_n$ tend vers $u_k$ mais elle est décroissante en $n$. Il y a un truc évident qui m'échappe ?

    Si quelqu'un voit...
  • Alternativement, tu peux utiliser un produit de Cauchy
  • Raoul a écrit:
    la suite $(u_k \frac{n}{n-k})_n$ tend vers $u_k$ mais elle est décroissante en $n$.

    Ah merde, je me suis trompé en regardant le sens de variation :)o. Merci d'être vigilant. Donc il faut trouver une meilleure justification (oui parce que les justifications justes sont meilleures que les fausses :-P). Je n'ai pas envie de réfléchir ce soir, donc je regarderai demain sauf si quelqu'un donne un bon argument d'ici là. Mais la limite est correcte ; ça j'en suis sûr (à 99%) parce c'est confirmé numériquement avec une bonne précision.

    sevaus : Tu réponds à qui ?
  • Ah OK, d'un côté je suis content car je commençais à m'inquiéter pour mes neurones... B-)-
  • Je répondais à raoul.S pour la démonstration de

    $\displaystyle \sum_{n = 0}^{+\infty} u_n x^n = e^{\text{Li}_2(x)}$
  • @sevaus je te remercie mais ma question portait sur un passage antérieur du calcul.
  • Bonjour raoul.S et Calli.

    La convergence monotone s'applique sur l'espace des suites avec la mesure de comptage.
    Les applications à intégrer sont les $f_{n}:k\mapsto\frac{u_{k}}{n-k}1_{\left\{ 0;\dots;n-1\right\} }\left(k\right)$. Elles ne sont pas monotones en $n$ car, pour $k<n$, on a $f_n(k)>f_{n+1}(k)$ alors que $f_n(n)=0$ et $f_{n+1}(n)=u_n>0$.

    Pour rectifier le tir, il faudrait envisager une majoration de toutes les $f_n$ par une même suite de somme finie et appliquer la convergence dominée.
  • Bonjour,

    Je considère acquis le fait que $\:\:\displaystyle \sum_{k=0}^{+\infty} u_k= \exp (\zeta(2)\:$) et reprends l'argument de Calli: il faut prouver que $\displaystyle \lim_{n\to + \infty} \left(\sum_{k=0}^{n-1} \dfrac{n u_k}{n-k}\right )= \sum_{k=0}^{+\infty} u_k. \qquad$
    $\displaystyle \sum_{k=0}^{n-1} \dfrac{n u_k}{n-k}= \sum_{k=0}^{n-1} u_k + \sum_{k=0}^{n-1} \dfrac {ku_k}{n-k}.\:\:\quad$Il suffit donc de s'assurer que $\displaystyle \lim_{n\to + \infty} \left(\sum_{k=0}^{n-1} \dfrac{k u_k}{n-k}\right )=0.\quad$ Pour cela:
    On démontre d'abord (avec une récurrence à partir de $n=3$) que; $\quad \forall n \in \N,\:\: nu_n \leqslant 2.$
    On déduit: $ \quad0 < nu_n< \dfrac 1n +2 \displaystyle \sum _{k=1}^{n-1}\dfrac 1{k(n-k)} = \dfrac 1n +\frac 4n \sum _{k=1}^{n-1}\dfrac 1{k} < \dfrac 1n + \dfrac 4n (1+\log n)$, ce qui entraîne l'existence de $\:\:A>0$ tel que:
    $\forall n \in \N,\:n>1:\:\:\: 0< nu_n< A \dfrac {\log n }n.\quad$ On obtient alors: $\:\:\displaystyle 0<\sum_{k=0}^{n-1} \dfrac{k u_k}{n-k}< \dfrac 2 {n-1} +A \sum_{k=2}^{n-1} \dfrac {\log k}{k(n-k)} <\dfrac 2 {n-1}+ \dfrac {2A \log n}n \sum_{k=2}^{n-1}\dfrac 1k.$
    Il vient: $\quad\displaystyle 0 <\sum_{k=0}^{n-1} \dfrac{k u_k}{n-k}< \dfrac 2 {n-1}+\dfrac {2A (\log n)^2}n ,\qquad \lim_{n\to + \infty} \left(\sum_{k=0}^{n-1} \dfrac{k u_k}{n-k}\right )=0.$
  • Bonjour à tous. Suite aux justifications (toujours brillantes !) de LOU16, il a été finalement montré que $\displaystyle \lim_{n\to + \infty}\Big(n^2u_n-\sum_{k=0}^{n-1} u_k\Big)=0$, et que d'autre part $\displaystyle \lim_{x\to 1}\sum_{k=0}^{+\infty} u_kx^k=\exp\big(\zeta(2)\big)$. J'ai l'impression qu'il manque un argument pour la convergence de la série numérique $\displaystyle \sum_{k\geq 0}u_k$ pour conclure. Non ?
  • Juste une remarque: si on sait déjà que la série des $u_n$ converge alors pas besoin de faire ce qu'a fait LOU16, mais il suffit d'appliquer la convergence dominée aux fonctions $f_n$ que j'ai définies dans mon précédent message.
  • Cet exercice est je pense un peu plus simple qu'il n'y paraît en commençant par poser $w_{n}=\frac{1}{n+1}$, et alors pour $\left|x\right|<1$ on a:
    \begin{align*}
    \underbrace{\Big(\sum_{n}u_{k}x^{k}\Big)}_{f\left(x\right)}\,\underbrace{\Big(\sum_{n}w_{k}x^{k}\Big)}_{g\left(x\right)} & =\sum_{n}\sum_{k=0}^{n}u_{k}w_{n-k}x^{n}\\
    & =\sum_{n}\sum_{k=0}^{n}\frac{u_{k}}{n+1-k}x^{n}\\
    & =\sum_{n}\left(n+1\right)u_{n+1}x^{n+1}=xf'\left(x\right)

    \end{align*} et le résultat découle de la résolution de cette équation différentielle et du fait que $$ -\int_{0}^{1}\frac{\ln\left(1-t\right)}{t}dt=\frac{\pi^{2}}{6}.$$
  • PS au message précédent : pour minorer le rayon de convergence de $f$, il suffit (récurrence immédiate) de voir par exemple que $u_n\leqslant n+1$ pour tout naturel $n$.
  • Merci LOU16. (tu)

    dedekind93 : Il y a un théorème qui dit que si $(u_k)$ est une suite positive et que la série entière $f :x\mapsto \sum\limits_{k=0}^\infty u_k x^k$ a un rayon de convergence supérieur ou égal à 1, alors $\sum\limits_{k=0}^\infty u_k = \lim\limits_{x\to1^-}\sum\limits_{k=0}^\infty u_k x^k$ dans $\overline{\Bbb R}$.

    troisqua : Il me semble que tu as oublié une multiplication par $n$ en posant $f_{n}:k\mapsto\frac{u_{k}}{n-k}1_{\left\{ 0;\dots;n-1\right\} }\left(k\right)$. Donc la seule domination que tu peux avoir c'est par $(ku_k)$ qui n'est pas sommable.
    troisqua a écrit:
    Cet exercice est je pense un peu plus simple qu'il n'y paraît

    C'est sûr qu'une fois qu'on a la solution, c'est facile.
  • Merci Calli : oui c'est ce que je pensais mais je me demandais s'il n'y avait pas un argument caché que j'aurais raté !
    troisqua : je ne comprends pas comment tu conclus. Il me semble que tu arrives -sans Fubini- à la formulation exponentielle trouvée initialement par Calli...et après ?
  • Calli : tu as raison pour l'oubli du produit par $n$. Bref, je pense que c'était une mauvaise piste de chercher à montrer la convergence de $n^2 u_n$ vers la somme de la série des $u_n$: trop compliqué.

    Sinon, je dis que cet exo était plus simple qu'il n'y paraissait.... modulo le fait qu'on repère la convolution qui figure dans la définition de $u_n$.
  • @troisquart: d'où vient le $x$ en plus entre $\sum_{k=0}^{n}\frac{u_{k}}{n+1-k}x^{n}$ et $\left(n+1\right)u_{n+1}x^{n+1}$ ?

    PS: j'ai bien compris d'où sortait le terme $(n+1)u_{n+1}$ par contre :-D
  • troisqua a écrit:
    je pense que c'était une mauvaise piste de chercher à montrer la convergence de $n^2 u_n$ vers la somme de la série des $u_n$: trop compliqué.

    La convergence de $n^2 u_n$ vers $\sum_n u_n =e^{\zeta(2)}$ est directement équivalente à l'équivalent qu'on veut trouver : $u_n\sim \frac{e^{\zeta(2)}}{n^2}$. Et en général montrer la convergence d'une suite est plus facile qu'un équivalent qui tend vers 0. Donc je ne vois pas comment éviter de montrer que $n^2 u_n \to \sum_n u_n$.
  • Un de mes élèves a eu un exercice très similaire à l'oral d'Ulm, il y a 2 ans.
    Il y avait un $n-k+2$ au dénominateur au lieu du $n-k+1$ et la question était de calculer la somme de la série $\displaystyle \sum_{n=0}^{+\infty}\frac{u_k}{2^k}$.

    L'introduction de la série entière et l'utilisation de la série produit de Cauchy et de l'équa diff sont effectivement les ingrédients qui permettent de s'en sortir.
  • @dedekind93.
    Je te propose une rédaction détaillée.
    Par une récurrence immédiate, pour tout naturel $n,$ on a $0\leqslant u_{n}\leqslant n+1$ si bien que $f\left(x\right)=\sum_{n=0}^{+\infty}u_{n}x^{n}$ a un rayon de convergence supérieure ou égal à $1$. On note $I=]-1;1[$ et pour $x\in I$ on a:
    \[
    -\ln\left(1-x\right)=\sum_{n=0}^{+\infty}\frac{x^{n+1}}{n+1}=x\sum_{n=0}^{+\infty}\frac{x^{n}}{n+1}
    \] et donc
    \begin{align*}
    -f\left(x\right)\ln\left(1-x\right) & =x\sum_{n=0}^{+\infty}u_{n}x^{n}\sum_{n=0}^{+\infty}\frac{x^{n}}{n+1}\\
    & =x\sum_{n=0}^{+\infty}\underbrace{\sum_{k=0}^{n}\frac{u_{k}}{n+1-k}}_{\left(n+1\right)u_{n+1}}x^{n}\\
    & =x\sum_{n=0}^{+\infty}\left(n+1\right)u_{n+1}x^{n}\\
    & =xf'\left(x\right)

    \end{align*} La fonction $x\mapsto\frac{-\ln\left(1-x\right)}{x}$ est continue sur $I$ et, pour une certaine primitive $F$ de $f$ sur $I$ on a $f\left(x\right)=e^{F\left(x\right)}$. Comme $f\left(0\right)=u_{0}=1$ on a $F\left(0\right)=0$. Donc $F\left(x\right)=\int_{0}^{x}\frac{-\ln\left(1-x\right)}{x}dx$ qui converge en $1^{-}$. Ainsi $f\left(x\right)=\sum_{n=0}^{+\infty}u_{n}x^{n}$ possède une limite finie en $1^{-}$. Donc, vu que les $u_{n}$ sont positifs, la série $\sum_{n=0}^{+\infty}u_{n}$ converge vers $f\left(1^{-}\right)$ (cf cette petite propriété). Or
    \begin{align*}
    F\left(1\right) & =\int_{0}^{1}\frac{-\ln\left(1-x\right)}{x}dx\\
    & =\int_{0}^{1}\sum_{n=0}^{+\infty}\frac{x^{n}}{n+1}dx\\
    & =\sum_{n=0}^{+\infty}\int_{0}^{1}\frac{x^{n}}{n+1}dx=\sum_{n=0}^{+\infty}\frac{1}{\left(n+1\right)^{2}}=\frac{\pi^{2}}{6}

    \end{align*} et finalement
    \[
    \sum_{n=0}^{+\infty}u_{n}=e^{F\left(1\right)}=e^{\frac{\pi^{2}}{6}}
    \]
    [édit: pour la détermination de l'équivalent de $u_n$, cf ce message un peu plus bas]
  • Merci troisqua. Ma question portait néanmoins davantage sur ta conclusion quant au but de l'exercice, à savoir un équivalent. Et là je ne vois pas quelque chose de plus simple que l'idée de Calli (rendue rigoureuse par LOU16) de partir de la relation de récurrence...
  • A noter que $n!u_{n}=\frac{A323290(n)}{A323291(n)}$
  • Dans la même veine, je vous soumets ceci :

    Soit $u_n$ une suite, $u_0=1$ et pour tout $n \in N^*, u_{n+1}= \sum_{k=0}^n u_k u_{n-k}$

    Trouver $u_n$ en fonction de $n$ .
  • Totem, c'est classique, ce sont ces bons vieux nombres de Catalan, la famille de nombres entiers qui a le plus d'interprétations combinatoires, et de loin.
  • Quelqu'un qui a déjà vu un produit de Cauchy (ce qui est inévitable en ayant lu ce fil) saura immédiatement comment partir.

    Personnellement j'ai toujours trouvé ça très beau que l'on puisse obtenir des informations sur certaines suites à l'aide de leurs séries génératrices (c'est sûrement pour ça que j'aime la théorie analytique des nombres, où les séries génératrices sont souvent des séries de Dirichlet :-D). On a l'impression d'une intervention divine de l'analyse dans un problème combinatoire ou algébrique.
  • @Poirot: intervention divine ?? rien que ça !! ::o:-D
    Je croyais que tout était "connecté" avec tout en maths...
  • D'accord avec Poirot et dans ce cadre d'idées P. Flajolet nous a laissé ce livre.
  • Pour revenir au problème initial on peut démontrer que la suite $(n^2u_n)$ a pour limite $\displaystyle\sum_{k=0}^{+\infty}u_k$ sans introduire de série entière. Cela utilise les mêmes arguments que la démonstration de LOU16 avec le résultat connu : $H_n=\displaystyle\sum_{k=1}^n\dfrac1k\sim\log n$.

    1) $\max(u_0,\dots,u_{n-1})\leq1$ entraîne que $u_n\leq\dfrac1n H_n\leq1$ donc $u_n\leq1$ pour tout $n$.

    2) $u_k\leq\dfrac1k H_k$ pour $k\geq1$ donne $n^2u_n\leq1+\displaystyle\sum_{k=1}^{n-1}\dfrac{nH_k}{k(n-k)}\leq1+H_{n-1}\displaystyle\sum_{k=1}^{n-1}\left(\dfrac1k+\dfrac1{n-k}\right)=1+2H_{n-1}^2\sim 2(\log n)^2$ donc la série $(\sum u_k)$ converge.

    3) $n^2u_n=\displaystyle\sum_{k=1}^{n-1}u_k+r_n$ avec $r_n=\displaystyle\sum_{k=1}^{n-1}\dfrac{ku_k}{n-k}\leq \sum_{k=1}^{n-1}\dfrac{1+2H_{k-1}^2}{k(n-k)}\leq (1+2H_{n-2}^2)\dfrac1n\displaystyle\sum_{k=1}^{n-1}\left(\dfrac1k+\dfrac1{n-k}\right)=\dfrac{2H_{n-1}(1+2H_{n-2}^2)}n\sim\dfrac{4(\log n)^3}n$ qui tend vers $0$.

    On a démontré que $u_n\sim\dfrac S{n^2}$ avec $S=\displaystyle\sum_{k=0}^{+\infty}u_k$.
    Pour le calcul de $S$ il faut bien sûr introduire une série entière.
  • @Dedekind,
    Effectivement mon message visait à justifier en détail la convergence de la série des $u_n$. Pour l'équivalent, on peut procéder ainsi:
    Soit $n> m$. On a
    \[
    \sum_{k\in m}\frac{nu_{k}}{n-k}\leqslant\underbrace{\sum_{k\in m}\frac{nu_{k}}{n-k}+\sum_{k=m}^{n-1}\frac{nu_{k}}{n-k}}_{n^{2}u_{n}}\leqslant\sum_{k\in m}\frac{nu_{k}}{n-k}+\frac{n}{n-m}\sum_{k=m}^{n-1}u_{k}
    \]
    Ainsi: $\sum_{k\in m}u_{k}\leqslant\underline{\lim}_{n}n^{2}u_{n}\leqslant\overline{\lim}_{n}n^{2}u_{n}\leqslant\sum_{k\in m}u_{k}+\sum_{k=m}^{+\infty}u_{k}$ et donc, à la limite en $m$ on obtient $n^{2}u_{n}\underset{n}{\longrightarrow}\sum_{m}u_{m}=e^{\frac{\pi^{2}}{6}}$ d'où l'équivalent souhaité.
  • @side: quand tu parles des "mêmes arguments" tu évoques ma proposition de solution ?
  • @side: j'ai proposé cette solution car j'ai vu que tu suivais ce fil :-) et on avait déjà discuté ensemble de la simplicité qu'apportaient les notions de limite supérieure et inférieure pour résoudre ce genre de problème. Il me semble qu'ici, c'est assez frappant.
  • [Toujours dans le HS sur les limites sup ou inf]
    Chaque fois que je repère qu'une somme doit être scindée, je démarre comme la preuve précédente, et la suite est quasi automatique si le fait que la suite devait être scindée était la clé de la preuve. C'est une question d'habitude je pense et les automatismes arrivent vite dès qu'on a vu deux ou trois fois ce principe en actions (par exemple pour les moyennes de Cesàro, ou pour la sommation d'équivalents). C'est aussi peut-être plus naturel quand on bosse beaucoup un cours de théorie de la mesure.
    [Fin du HS]
  • troisqua a écrit:
    $$\sum_{k\in m}\frac{nu_{k}}{n-k}\leqslant\underbrace{\sum_{k\in m}\frac{nu_{k}}{n-k}+\sum_{k=m}^{n-1}\frac{nu_{k}}{n-k}}_{n^{2}u_{n}}\leqslant\sum_{k\in m}\frac{nu_{k}}{n-k}+\color{red}{\frac{n}{n-m}}\sum_{k=m}^{n-1}u_{k}$$
    Ainsi: $\sum_{k\in m}u_{k}\leqslant\underline{\lim}_{n}n^{2}u_{n}\leqslant\overline{\lim}_{n}n^{2}u_{n}\leqslant\sum_{k\in m}u_{k}+\sum_{k=m}^{+\infty}u_{k}$.

    Non, en rouge ça doit être $n$, donc ça ne marche pas. Franchement, est-ce que Lou aurait fait tout ça, si c'était si simple de prouver $n^{2}u_{n} \to \sum_{k=0}^{+\infty}u_{k}$ ? Parfois quand on tombe sur une méthode qui a l'air "trop" facile, il faut se demander s'il n'y a pas une erreur.
Connectez-vous ou Inscrivez-vous pour répondre.